7a3f495248d2f8438617dd2abd5fd4504ad7afe0
[course.git] / latex / problems / Serway_and_Jewett_4 / problem23.12.tex
1 \begin{problem*}{23.12}
2 Consider the arrangement shown in Figure P23.12.  Assume that $R =
3 6.00\Omega$, $l = 1.20\U{m}$, and a uniform $B=2.50\U{T}$ magnetic
4 field is directed into the page.  At what speed should the bar be
5 moved to produce a current of $0.500\U{A}$ in the resistor.
6 \end{problem*}
7
8 \begin{solution}
9 This problem is almost identical to the recitation Problem 13.
10 Copying the induced current formula:
11 \begin{equation}
12  I = \frac{\varepsilon}{R} = \frac{-lvB}{R} \;,
13 \end{equation}
14 where the $-$ sign indicates the current is counterclockwise (out of the
15 page), so current flows upward through the bar.
16
17 We can solve this equation for $v$, yeilding
18 \begin{equation}
19  v = \frac{IR}{lB} = \ans{1.00\U{m/s}} \;.
20 \end{equation}
21 \end{solution}